LSAT and Law School Admissions Forum

Get expert LSAT preparation and law school admissions advice from PowerScore Test Preparation.

User avatar
 Dave Killoran
PowerScore Staff
  • PowerScore Staff
  • Posts: 5848
  • Joined: Mar 25, 2011
|
#71208
This game is discussed in our Podcast: LSAT Podcast Episode 31: The September 2019 LSAT Logic Games Section

Complete Question Explanation
(The complete setup for this game can be found here: https://forum.powerscore.com/lsat/viewtopic.php?t=31299)

The correct answer choice is (E).

If F and J are displayed next to each other, then we must be in the 3 kittens/2 puppies scenario (since there are kittens displayed first and last, and a block would have to be 1-2 or 4-5). Thus, our choices for the kittens are FJ in a block, and then the choice of G or H. However, since FJ are next to each other in 1-2 or 4-5, the remainder kitten must be in 5 or 1. Consequently, G could never be displayed second, and thus via the contrapositive of the fourth rule we can conclude that W cannot be included in the display. Thus, answer choice (E) is correct.

Note that T also cannot be included in this display, but the test makers chose to go a level deeper here and did not offer T as an option.


Answer choice (E): This is the correct answer choice.
 hannahmag
  • Posts: 1
  • Joined: Nov 20, 2019
|
#72208
From this question I deduced that with F and J, you would have F, J and either G or H, so 3 kittens which means 2 puppies. With 2 puppies, T cannot be included because T requires 3 puppies. But I couldn't go from there. Could you tell me how to approach this question, and how W is the answer?
 dyogenes
  • Posts: 7
  • Joined: Mar 11, 2019
|
#72237
hannahmag wrote:From this question I deduced that with F and J, you would have F, J and either G or H, so 3 kittens which means 2 puppies. With 2 puppies, T cannot be included because T requires 3 puppies. But I couldn't go from there. Could you tell me how to approach this question, and how W is the answer?

I originally got stuck at T-out too, cause it felt like a sufficient inference for the fourth question of the section. Took me a (very :( ) long time to notice that no matter how you place the FJ block and G/H, you can never arrive at G occupying 2, which means W must be out.

Hope this helps!
 Jeremy Press
PowerScore Staff
  • PowerScore Staff
  • Posts: 1000
  • Joined: Jun 12, 2017
|
#72263
Hi Hannah,

The answer Dyogenes gave is perfect.

If you're stuck, particularly on a CANNOT be true question like this one, look for a variable in the answer choices that is likely to have a strong "constraining" effect on the solution (i.e. a variable that forces you to make additional placements), such that it might not be possible to use it. R and S are "random" variables in the scenario, so they don't have a very strong effect on the other variables (and it's impossible one would have an impact the other one wouldn't have). So "softly eliminate" those two. In this question, G and H are essentially interchangeable variables: you already know that one is and one is out. But the choice as to which you leave in and which you leave out doesn't have much effect on the other variables. W is the only variable that has a strong effect in this situation. If it's in, it forces you to do something with G. That "something" is something you won't be able to do, given the placement requirement of F and J. So that's what forces W out. Tricky question, for sure, but you can sometimes predict a likely CANNOT answer using the above insights.

I hope this helps!

Jeremy

Get the most out of your LSAT Prep Plus subscription.

Analyze and track your performance with our Testing and Analytics Package.